Ice tea

This topic has expert replies
Legendary Member
Posts: 510
Joined: Thu Aug 07, 2014 2:24 am
Thanked: 3 times
Followed by:5 members

Ice tea

by j_shreyans » Mon Sep 29, 2014 9:20 pm
A convenience store manager noticed that a cooler which had been stocked with only a cola product and an iced-tea product had 15 colas left at the end of the day but only 3 iced-tea beverages. As a result, the manager reasoned that he should increase the amount of iced tea and decrease the amount of cola he ordered from the distributor.

Which of the following, if true, would most strengthen the manager's rationale for ordering more iced tea and less cola?

A) The cooler in question is the only place in the store where the cola and iced tea beverages are stocked.

B)On that day, a month-long $1,000,000 sweepstakes began, with prizes awarded via the bottlecaps on the iced tea beverage.

C)At the beginning of the day, the cooler was stocked with at least as many of the iced tea beverages as of the cola beverages.

D)On the subsequent day, the remaining three iced tea beverages all sold within the first hour after the store opened.

E)During that week, a special "buy one, get one free" sale was in effect for the cola beverage.

Guys ,

why not D why C

Can you guys pls explain each options.

GMAT/MBA Expert

Master | Next Rank: 500 Posts
Posts: 147
Joined: Mon Jul 21, 2014 3:52 pm
Thanked: 53 times
Followed by:21 members

by David@GMATPrepNow » Tue Sep 30, 2014 3:21 pm
Hi j_shreyans,

It comes down to an absurdist argument to prove that C is correct. Here goes...

What if the cooler was stocked in the morning with 16 iced tea beverages, and 15 TRILLION cola beverages? The point is, no matter how many of either beverage was stocked, we could still have the result that the passage outlines, namely that there were 1 colas left, and 3 iced teas left at the end of the day.

We're looking for an answer that strengthens the rationale for ordering more iced tea and less cola. Answer C does that nicely.

Legendary Member
Posts: 510
Joined: Thu Aug 07, 2014 2:24 am
Thanked: 3 times
Followed by:5 members

by j_shreyans » Tue Sep 30, 2014 10:13 pm
Hi David ,

This is strenght question so we have to do the same.

Here the" manager reasoned that he should increase the amount the ice tea and decrease the amount of cola" so have we to find the answer that strength the manager reasoned right?

But in option C says At the beginning of the day, the cooler was stocked with at least as many of the iced tea beverages as of the cola beverages. and i think this is not strengthening the manager's reasoned.

Option D is strengthening i guess.

Pls correct me if i am wrong really need a help here.

User avatar
Senior | Next Rank: 100 Posts
Posts: 30
Joined: Fri Jan 30, 2009 12:51 am
Location: Pune

by ameya85 » Wed Oct 01, 2014 1:42 am
Hi j_shreyans,

I think D does not strengthen author's argument. Partly, I find D to be incomplete. First, we say all 3 ice tea beverages were sold within one hour, but we have no information on how many cola beverages are sold. Let us say on subsequent day 3 ice tea and 14 cola beverages were sold. In this case, D does not strengthen the argument.

On the otherhand, C does strengthen the argument.Simply because we know that we started with equal number of beverages for both ice tea and cola and at the end of the day, we are left with only 3 ice tea beverages.

This is similar to saying that you and your friend started a race and you completed the race 3 hours before your friend did. In this case, can you assume that your friend was slow? No, it could be possible that you got a head start and that is why you reached early. If you know that you and your friend started at same point, and then you completed the race before your friend, you can safely assume that your friend was indeed slow.

I hope I did not mess it up!

Ameya

GMAT/MBA Expert

Master | Next Rank: 500 Posts
Posts: 147
Joined: Mon Jul 21, 2014 3:52 pm
Thanked: 53 times
Followed by:21 members

by David@GMATPrepNow » Wed Oct 01, 2014 5:02 am
Nice analogy Ameya!

As you noted, j_shreyans, we want to strengthen the author's argument. As Ameya points out, D does not actually strengthen the argument. In truth, we don't know what will happen over the course of the day - it could be that three iced tea beverages all sold within the first hour after the store opened, but we don't know what will happen during the rest of the day. My point is that selling the remaining three iced tea beverages in the first hour is not necessarily a good predictor of what will happen during the rest of the day.

User avatar
GMAT Instructor
Posts: 2193
Joined: Mon Feb 22, 2010 6:30 pm
Location: Vermont and Boston, MA
Thanked: 1186 times
Followed by:512 members
GMAT Score:770

by David@VeritasPrep » Wed Oct 01, 2014 4:53 pm
Showing that answer choice C is correct is a very simple matter.

This is like a quantitative problem with two numbers that may not be comparable.

The manager is basing his recommendation of ordering more iced tea on the basis of the number of drinks left at the end of the day. He or she is using the number of colas and teas left to give a rough idea of which one sells better. It seems reasonable that if there are only three teas left that the tea is selling better.

But what if there were three teas at the beginning of the day? What if those same three teas have been there for a week. Clearly tea is not a big seller.

Under my hypothetical situation choice D does not help, because even if the three teas sold the next day, if those are the only three teas sold that week it does not follow that the manager should increase the tea and decrease the cola order.

On the other hand, if choice C is true and if the cooler was equally stocked at the beginning of the day, then it appears that tea is outselling cola by at least 12 units. This means that the managers rough observation that the tea is selling better is strengthened by the fact that it is now comparable. Of course without choice C it may be true that the case had 30 more colas than teas at the beginning of the day. In which case 12 more colas left over means that colas outsold teas by 18...so choice C strengthens things and the absence of choice C calls the decision into question.

I hope that clarifies!
Veritas Prep | GMAT Instructor

Veritas Prep Reviews
Save $100 off any live Veritas Prep GMAT Course

Moderator
Posts: 426
Joined: Tue Aug 22, 2017 8:48 pm
Followed by:1 members

by BTGmoderatorAT » Wed Sep 06, 2017 5:24 am
I believed it is letter "C"

Based on the passage there's no mentioning on the quantity of cola & iced-tea stocked. We cannot conclude which of the 2 products are saleable. Am I correct?